Equations in prime powers (Q1068192)

From MaRDI portal
scientific article
Language Label Description Also known as
English
Equations in prime powers
scientific article

    Statements

    Equations in prime powers (English)
    0 references
    0 references
    0 references
    0 references
    0 references
    1985
    0 references
    There are some number theoretical questions as Hauptidealsatz, Kronecker classes or arithmetically equivalent fields [c.f. the second author and \textit{D. B. Wales}, J. Algebra 96, 94-113 (1985; Zbl 0577.20005)] where problems of algebraic number theory are translated into group theoretical properties. So it is not surprising that scholars of group theory attempt to investigate number theoretical problems. The authors consider the diophantine equation (*) \((q^ n-1)/(q-1)=p^ a\) with positive integers \(a\), \(q\), \(n\), \(q\geq 2\), \(n\geq 3\), and \(p\) prime. Applying Zsigmondy's theorem on primitive prime divisors it is an exercise to see that \(n\) is prime and if \(q=r^ m\), then \(m\) is a power of \(n\), \(p\equiv 1\pmod{mn}\) [c.f. the reviewer's thesis, Diophantische Probleme bei Kreisteilungspolynomen, FU Berlin (1975), p. 40]. Theorem 1 shows that \((n,a)=1\) if \(r\) is a prime. The other number theoretical results are mainly known (Nagell, Ljunggren, Schinzel) or erroneous, e.g. the class number of \({\mathbb{Q}}(\sqrt{-341})\) is 28 [\textit{Z. I. Borevich} and \textit{I. R. Shafarevich}, Zahlentheorie (1964; Zbl 0121.042), Tabelle 4], so the requirements of corollary 2 are fulfilled, but the equation \(y^ 5=x^ 2+341\) has the solution \(y=377\), \(x=2.759.646\) [c.f. \textit{J. Blass}, Math. Comput. 30, 638-640 (1976; Zbl 0336.10012)]. The error seems to be in Theorem 4 where 331 perhaps should be 341. \{Reviewer's remark: According to the table in Borevich-Shafarevich there would be another counterexample: \(y^ 5=x^ 2+485\) has the solution \(y=9\), \(x=242\). But here the table is wrong: The class number of \({\mathbb{Q}}(\sqrt{-485})\) is 20, not 16 as indicated. -- Friendly communicated by J. Antoniadis.\} Finally the authors sketch a short proof that for a given prime p there are only finitely many a, q, n, \(n\geq 3\), with (*) [c.f. the more general result of \textit{T. Shorey} and \textit{R. Tijdeman}, Math. Scand. 39, 5-18 (1976; Zbl 0341.10017)]. From this they deduce corollary 3: If \(p\) is an odd prime, there are only finitely many simple groups which have a \(p\)-complement (i.e., a subgroup of order prime to \(p\) and of index a power of \(p\)).
    0 references
    0 references
    0 references
    0 references
    0 references
    0 references
    0 references
    quintic equation
    0 references
    diophantine equation
    0 references
    class number
    0 references
    simple groups
    0 references
    \(p\)-complements
    0 references
    0 references
    0 references